Solving the logistic differential equation part 1 | Khan Academy

แชร์
ฝัง
  • เผยแพร่เมื่อ 30 ก.ย. 2024

ความคิดเห็น • 39

  • @zhaoyangxie4396
    @zhaoyangxie4396 4 ปีที่แล้ว +9

    Great video. Just have one question: why don't you use separation way just like what you did in the previous videos? Thanks.

    • @Syndicalism
      @Syndicalism 2 ปีที่แล้ว +2

      I know this is a year late reply but he does use separation of variables. First thing he does. He then does partial fraction decomposition to help find the anti derivative on the LHS of the equation.

  • @rebeccadoorley83
    @rebeccadoorley83 8 ปีที่แล้ว +11

    was completely with you until 7.20m havent a clue after that!

    • @marioleon4128
      @marioleon4128 5 ปีที่แล้ว +5

      pretty much he linearized the whole integral that was created by using partial fractions and broke that big integral up into two smaller ones like this: 1/n + (1/k)/(1-n/k)dn/dt = r
      Is the same as ↳ ∫1/n +(1/k)/(1-n/k)dn = ∫r dt

    • @randomeggthatworksforthefb7172
      @randomeggthatworksforthefb7172 3 ปีที่แล้ว +1

      @@marioleon4128 I did help! even though this is two years later haha

  • @13identity
    @13identity 9 ปีที่แล้ว +18

    Holy cow this is loooong and complicated.

  • @meltemkaya9725
    @meltemkaya9725 4 ปีที่แล้ว +4

    if we regulate 1/(1-N/K) as K/(K-N) then we can see K/N(K-N) = 1/N - (-1)/(K-N). And the solution will be easier.

    • @dibehemoth401
      @dibehemoth401 2 ปีที่แล้ว

      Yeah. The problem can be solved way faster this way.

    • @ipeklacin2918
      @ipeklacin2918 ปีที่แล้ว

      helal meltem as bayrakları

  • @BiscuitZombies
    @BiscuitZombies 4 ปีที่แล้ว +3

    11:15 The integral of (-1/k)/(1-n/k) with respect to n is 1/k * Ln(1-n/k), note the 1/k coefficient. Why does yours not have a coefficient?

    • @zareefamyeen6925
      @zareefamyeen6925 4 ปีที่แล้ว +1

      The integral of (-1/k)/(1-n/k) IS -ln(1-n/k). If you use u = 1-n/k you should get integral of - du/u = - lnu

    • @athenovae
      @athenovae 5 วันที่ผ่านมา

      I was just about to say. He’s missing it.

    • @BiscuitZombies
      @BiscuitZombies 5 วันที่ผ่านมา

      @@athenovae I have since realised that what I said is wrong. the 1/(1-n/k) produces a factor of k at the front. This cancels the -1/k.

  • @athenovae
    @athenovae 5 วันที่ผ่านมา

    11:21 I think you’re missing 1/k in front of the second ln.

  • @peter_roth_8812
    @peter_roth_8812 10 ปีที่แล้ว +2

    Geat job how you did this partial fractions expansion.

    • @stevenson720
      @stevenson720 7 ปีที่แล้ว

      Hans Peter Roth
      yep. I had completely forgotten that was a thing

  • @Wondermass
    @Wondermass 6 ปีที่แล้ว

    We did a video (featuring the logistic equation) about where equations come from in social science: th-cam.com/video/zXY6LEwHgaI/w-d-xo.html

  • @tauseefahmedmemon9148
    @tauseefahmedmemon9148 6 ปีที่แล้ว

    how is he taking derivative of terms with no "t" in them w.r.t "t" and not making them 0???

  • @ConnorMooneyhan1
    @ConnorMooneyhan1 7 ปีที่แล้ว +4

    At 7:30, I'm very confused. Why are we taking the antiderivative with respect to N and then taking the antiderivative of that with respect to t?

    • @NazmusLabs
      @NazmusLabs 7 ปีที่แล้ว

      Connor Mooneyhan he wasn't. He was just showing you that you CAN take antiderivatevs with respect to t as well because T is a dependent variable of N. He showed you both options.

  • @prosimion
    @prosimion ปีที่แล้ว

    thank you once again sir....sorry I haven't said that in a while.

  • @lyfxyz12
    @lyfxyz12 9 ปีที่แล้ว +2

    goooood video

  • @rogelioherbolingo2843
    @rogelioherbolingo2843 2 ปีที่แล้ว

    Is separable equation with an implicit or explicit solution not applicable in this situation??

  • @Clin270
    @Clin270 4 ปีที่แล้ว +1

    1337, nice

  • @hrushikeshmandekar6572
    @hrushikeshmandekar6572 3 ปีที่แล้ว +1

    Thanks, it helps me lot.

  • @dialaali9510
    @dialaali9510 3 ปีที่แล้ว

    can you help me with (computing the poincare map )?
    and thank you for this video its helpful

  • @mohbada1134
    @mohbada1134 4 ปีที่แล้ว

    What is demographic explanation of K

  • @bobabooks
    @bobabooks หลายเดือนก่อน

    nft haha

  • @dragonbrave8
    @dragonbrave8 10 ปีที่แล้ว

    mathematics is so versatile, is like water taking the shape of its vessel.
    Im stuck at 8:44, I don't get that Ln derivative.

  • @YouUndeground
    @YouUndeground 5 ปีที่แล้ว +1

    Thank you!

  • @gexwing
    @gexwing 10 ปีที่แล้ว

    haha part 1 isn't showing in my inbox, yet here it is :)

  • @nullheim
    @nullheim 9 ปีที่แล้ว +4

    Well at 6:33, I didnt use the partial fraction like you, I used 1/(K-N) instead of using (1/K)/(1-(N/K)). Here is the problem ; mathematically they are exatly the same, but I got different answer, my calculations are right just like yours. Aren't they same expressions, if not, can you tell me why not? I found the equation as follows :
    N(t)=((KN.)/(e^(-rt)K+N.)) yours is N(t)=((KN.)/((K-N.)e^(-rt)+N.)) only different thing between our functions is you have coefficent (K-N.) to e^(-rt), mine has a coefficient K

    • @prabinlamsal5125
      @prabinlamsal5125 4 ปีที่แล้ว +1

      Facing the same problem after more than 4 years man.

  • @StainlessSteelFaucet
    @StainlessSteelFaucet 4 ปีที่แล้ว

    I'm in pain

  • @grozkid
    @grozkid 7 ปีที่แล้ว +9

    terrible

  • @ShiroWretchedEggX
    @ShiroWretchedEggX 9 ปีที่แล้ว

    Much easier to solve if you know how to solve bernoulli equations.

  • @Catadventures131
    @Catadventures131 6 ปีที่แล้ว +2

    Calculus sucks